LSAT and Law School Admissions Forum

Get expert LSAT preparation and law school admissions advice from PowerScore Test Preparation.

 ellenb
  • Posts: 260
  • Joined: Oct 22, 2012
|
#10317
Dear Powerscore,

For this question I thought that the correct answer will be B, however, it is C. It did not make sense why C is the right answer. I thought that, the revcenues that they will generate have to be comparable to the revenues that they would generate from the essential goods. Please let me know where I went wrong.

Thanks

Ellen
 BethRibet
PowerScore Staff
  • PowerScore Staff
  • Posts: 200
  • Joined: Oct 17, 2012
|
#10363
Hi Ellenb,

Thanks for your question! The critical element in the stimulus is this phrase: "this tax will result in a substantial increase in government revenues". For this statement to prove correct, it will need to be true that when the proposed luxury tax is implemented, the people who buy those items don't stop buying. If they dislike the tax enough to forgo their purchases, then they won't be paying the tax, and they won't increase government revenues. The assumption in the officials' argument is that the sales from luxury goods will be enough generate enough sales tax to result in a "substantial increase in government revenues".

Answer choice B asks: "Will the revenues generated by the proposed tax be comparable to those that are currently being generated by taxes on essential goods?"

We actually don't need the answer to this question to evaluate whether the luxury tax will generate a "substantial increase" in revenues -- because the argument is not claiming that the luxury tax will generate more or less revenue than a tax on essential goods, only that it will cause a substantial increase in revenue coming in to the government.

Hope that helps!
Beth
 ellenb
  • Posts: 260
  • Joined: Oct 22, 2012
|
#10366
So, for instance I could have $ 5000 generated by essential goods and if I pick answer choice b than they want to say I would have $5000, which actually does not matter, since what matters that I need to have substantial increase in the government and substantial might be 10 times more than what we would make based on esential goods. So than what mmatters is whether ppl will be buying the luxury goods so we can have revenues. Did I understand it correctly?

Thanks

Ellen
 Sherry001
  • Posts: 81
  • Joined: Aug 18, 2014
|
#21205
Hello I had trouble agreeing with the right answer. Could you please explain this one to me? Here's my reasoning!

1- the traditional taxation system turned low and mid income tax payers against the government .
2- the new system will result into substantial increase
3- will only affect the wealthy

C: government officials have proposed adding a ne tax on purchases of luxury items such as yachts, private planes, jewels and furs.

My pre phrase was okay,well what if these rich people get pissed and not purchase these items. Then the proposal is screwed !

A) was very tricky .. But I eliminated this because higher rates I thought wouldn't necessarily lead to substantial increase if people don't buy!

B) my asking yes or no didn't d story this , so I eliminated it.

C) so I can see why asking yes.. Strengthens this. Because it leads to more people buying and having that substantial increase. But when I asked NO .. I felt it didn't weaken the argument. Because it doesn't have to lead to the current rate in sales.why can't it be more ?
I just felt if this is true then so should b!

D) we don't care about support as long they don't turn against the government.
E)irrelevant who cares who buys what ..as long as it leads to that substantial increase.


Thanks so much !
 Lucas Moreau
PowerScore Staff
  • PowerScore Staff
  • Posts: 216
  • Joined: Dec 13, 2012
|
#21219
Hello, Sherry,

Your logic is essentially sound. :)

A is no good because the relative tax rates aren't important to the conclusion, yes.
B doesn't matter - we're not talking about current tax rates on essential goods.
C is better than B, because C has to do more directly with the conclusion - that the new tax will provide a substantial revenue increase without affecting the middle class. If C isn't true, and the wealthy buy less luxury items, then the revenue increase may not materialize.
You are correct about D and E. :)

Hope that helps,
Lucas Moreau
 dbpk
  • Posts: 16
  • Joined: May 07, 2017
|
#37792
Lucas Moreau wrote:Hello, Sherry,

C is better than B, because C has to do more directly with the conclusion - that the new tax will provide a substantial revenue increase without affecting the middle class. If C isn't true, and the wealthy buy less luxury items, then the revenue increase may not materialize.
Using the variance test, I completely agree that answering "no, the sale of luxury items will not occur at the same rate under the tax" seriously weakens the official's argument. But answering "yes, the sale of luxury items WILL occur at the same rate under the tax" doesn't guarantee any support for the argument. It may be the case that there may be an increase in government revenue if luxury items sell at the same rate but not necessarily a substantial one.

How definitively should the variance test prove/weaken the argument to know the answer choice is correct?
Thank you very much!
User avatar
 Dave Killoran
PowerScore Staff
  • PowerScore Staff
  • Posts: 5981
  • Joined: Mar 25, 2011
|
#37800
Hi D,

Thanks for the question! Actually, that will support the government officials’ prediction because it shows that the basic assumption they are using about sales—that they will stay at least stable—is correct. Does it prove the argument? No, but the goal here is simply to support the idea under this side of the Variance Test. While proving it would be nice, that is not the requirement.

Secondarily, you mention that "... doesn't guarantee any support for the argument. It may be the case that there may be an increase in government revenue..." That increase in revenue is actually some support, just not complete and undeniable proof.

It looks to me like you fully understood this problem, but just didn't like the fact that (C) on the strengthen side didn't prove the officials' claim. That's ok as long as it would help at least a bit.

Please let me know if that helps. Thanks!
 dbpk
  • Posts: 16
  • Joined: May 07, 2017
|
#37822
This was very helpful! Thank you so much I appreciate your response!
 harvoolio
  • Posts: 62
  • Joined: Apr 25, 2018
|
#45840
I got this answer wrong for the precise reasoning Sherry has stated above. I pre-phrased if sales go down then this would affect the accuracy of the prediction so I looked for a question like "Will rich people and corporations decrease spending on luxury items?" If the answer is no the stimulus is strengthened, but if the answer is yes the stimulus is weakened.

So, I applied (C) and felt, like Sherry, that if the answer is no the stimulus is strengthened, but if the answer is yes the effect on the stimulus could either strengthen or weaken it depending on whether the change in current rates is an increase or a decrease. So, I eliminated the answer per the Variance Test because the polar opposites did not respectively weaken and strengthen the stimulus.

I also listened to Dave's explanation on the Advanced Logical Reasoning course but feel he is presuming that the sales go down and then arguing (correctly) that because sales goes down then the argument is weakened which I concur with entirely if we assume sales go down which the answer does not indicate. If, however, these rich people are buying Veblen luxury goods whose quantity demanded increases as price increases (original price plus new tax) and their spending changes from the current rate then the argument is strengthened and not weakened. Furthermore, a previous LSAC question that Jon discussed earlier in the Advanced Logical Reasoning course on exclusivity begins with a similar premise "Certain items—those with that hard-to-define quality called exclusivity—have the odd property, when they become available for sale, of selling rapid rapidly even though they are extremely expensive." So the idea that demand can go up if price goes up is not entirely foreign within Economics or to LSAC.

I was unclear about (A) but selected it because I was clear (C) failed the Variance Test (as I understand it). Also, for other strengthen or weaken LSAT questions I feel we regularly eliminate answers like (C) when the direction is unspecified so hence the impact is ambiguous. And if the answer is that I am to assume that a tax would cause demand to go down, I feel this opens up Pandora's Box/Slippery Slope. Where do I draw the line at inferring additional information that the answer choices do not state?

Or is my understanding of the Variance Test incorrect and that polar opposites do not have to produce respective weaken and strengthen answers? If that criteria is eliminated and I merely asked is the answer to the question "Will sales ... occur at current rates once the proposed tax ... has been passed?" relevant in evaluating the accuracy of the government officials' position? then the answer is Yes because if the answer is Yes then I know substantial revenue will be generated (assuming no tax evasion); if the answer is No I am then uncertain about whether substantial revenue will be generated (could be even greater revenue or less revenue).

Thanks in advance for a long post.

UPDATED - After sleeping on it there might be a way to see a change in sales (besides a decrease in sales) as also weakening the argument. Without a change in sales we are certain the tax will generate substantial revenue (assuming no tax evasion). Let's assume a change in sales had a 50% chance of an increase in sales and a 50% chance of a decrease in sales, this fact would still weaken the conclusion because instead of there being certainty that substantial revenue will be generated there is now a chance somewhere between a 0% and 50% (because a slight decrease in sales could still generate substantial revenues) of the official's prediction of substantial revenues being inaccurate.
 James Finch
PowerScore Staff
  • PowerScore Staff
  • Posts: 943
  • Joined: Sep 06, 2017
|
#46257
Hi Harvoolio,

This question is a bit tricky because the Variance Test for answer choice (C) actually has 3 answers, rather than 2: if we answer Yes, sales will remain the same, that clearly strengthens the argument. But if we answer No, then we also need to think about what possibilities are inherent in that answer. So we need to test both "No, sales will actually go up" (unlikely but possible) and "No, sales will effectively end, destroying the market for these luxury." In that case, sales going up again strengthens the argument, but sales going down to effectively nothing would weaken both prongs of the conclusion: no sales means no taxes, so no "substantial increase in government revenues" and the ending of a market for a good potentially meaning the ending of the industry that makes that good, which would presumably affect more people than "only the wealthy individuals and corporations" buying said good.

Hope this clears things up!

Get the most out of your LSAT Prep Plus subscription.

Analyze and track your performance with our Testing and Analytics Package.